Existe una versión más primitiva y natural del lema de Nakayama:
Dejemos que $R$ sea un anillo conmutativo, y $M$ sea una entidad finitamente generada $R$ -módulo y $I\subset R$ ser un ideal. Si $IM=M$ entonces $\exists f\in I,\forall m\in M,f\cdot m=m$ .
La prueba es un poco más difícil que la versión anterior, pero esto dice más o menos "Si $IM=M$ entonces uno de los elementos de $I$ estabiliza todos los elementos de $M$ ." Y creo que esto es mucho más natural que la versión anterior. Asumiendo esta versión, entonces podemos ver que la condición de "ideal de Jacobson" sólo se utiliza para mostrar que " $1-f$ es una unidad" que lleva a " $M=0$ ".
Para completar, doy una prueba de la versión anterior aquí:
Prueba: Dejemos que $m_1,...,m_n$ sea un conjunto de generadores de $M$ . Desde $IM=M$ tenemos una matriz $A'=(a_{ij})\in \textrm{M}_{n\times n}(R)$ tal que $A'\underline{m}=\underline{m}$ donde $\underline{m}=(m_1,...,m_n)^t$ y $a_{ij}\in I,\forall i,j\leq n$ . Ahora dejemos que $A=\textrm{Id}-A'$ por lo que tenemos $A\underline{m}=\underline{0}$ .
Tenga en cuenta que $\det A\in R$ está bien definida y existe una matriz $B\in \textrm{M}_{n\times n}(R)$ s.t. $BA=AB=\det A\cdot \textrm{Id}$ La prueba se encuentra en el Corolario 9.161 del libro Advanced Modern Algebra de Joseph J. Rotman. También $\det$ conmuta con el mapa cociente $R\rightarrow R/I$ con $A\equiv \textrm{Id} \ (\textrm{mod }I)$ podemos deducir que $\det A\in 1+I$ Así que $\exists f\in I$ , s.t. $\det A=1-f$ .
Para mostrar $f$ estabiliza $M$ basta con demostrar que $f\cdot m_i=m_i,\forall i$ . $$\underline{0}=BA\underline{m}=\det A\cdot \underline{m}=(1-f)\cdot \underline{m}$$ Así que $$f\cdot \underline{m}=\underline{m}$$ El resultado es el siguiente. Q.E.D
0 votos
¿Ha leído el artículo de Wikipedia?
1 votos
@Qiaochu Yuan - Lo he mirado, pero no parece haber ninguna intuición real dada allí (al menos ninguna que yo pueda ver).
2 votos
Esta pregunta acaba de ser referenciada en MathOverflow.Net - ¡puede que quieras echar un vistazo a las respuestas! mathoverflow.net/questions/61446/